Absolute Convergent, Conditionally Convergent?

In summary: That was a great explanation! This method definitely seems simpler and more efficient. In summary, the sequence ##\lim _{n\rightarrow \infty }\left( \dfrac {1} {\ln n}\right) ^{\dfrac {n} {n}}## simplifies to 0 and the root test can be used to show absolute convergence of the series.
  • #1
knowLittle
312
3

Homework Statement


##\sum _{n=2}\dfrac {\left( -1\right) ^{n}} {\left( \ln n\right) ^{n}}##

The Attempt at a Solution


I have applied the Alternating Series test and it shows that it is convergent. However, I need to show that it's either absolute conv. or conditionally conv.

Next, I tried the root test:
##\lim _{n\rightarrow \infty }\sqrt {\left| \left( \dfrac {1} {\ln n}\right) ^{n}\right| }##, **Correction; this is the root of n**
Now, I'm tempted to use direct comparison with 1/n harmonic series and show divergence. However, I don't know, if I am allowed to do this since the root test tells me that then I have to find the limit of my An and classify it accordingly.
According to the root test:
if the limit >1 or infinity it diverges
if the limit <1 it converges absolutely.
if the limit =1 it's inconclusive.
 
Physics news on Phys.org
  • #2
knowLittle said:

Homework Statement


##\sum _{n=2}\dfrac {\left( -1\right) ^{n}} {\left( \ln n\right) ^{n}}##


The Attempt at a Solution


I have applied the Alternating Series test and it shows that it is convergent. However, I need to show that it's either absolute conv. or conditionally conv.

Next, I tried the root test:
##\lim _{n\rightarrow \infty }\sqrt {\left| \left( \dfrac {1} {\ln n}\right) ^{n}\right| }##, **Correction; this is the root of n**
Now, I'm tempted to use direct comparison with 1/n harmonic series and show divergence. However, I don't know, if I am allowed to do this since the root test tells me that then I have to find the limit of my An and classify it accordingly.
According to the root test:
if the limit >1 or infinity it diverges
if the limit <1 it converges absolutely.
if the limit =1 it's inconclusive.

What do you get when you simplify$$
\lim_{n\rightarrow\infty}\root\scriptstyle ^n\of {\frac 1 {(\ln n)^n}}$$Then what happens as ##n\rightarrow \infty##? Don't answer that by talking about some series test. This is just the limit of a sequence. What do you get?
 
  • #3
LCKurtz said:
What do you get when you simplify$$
\lim_{n\rightarrow\infty}\root\scriptstyle ^n\of {\frac 1 {(\ln n)^n}}$$Then what happens as ##n\rightarrow \infty##? Don't answer that by talking about some series test. This is just the limit of a sequence. What do you get?

$$
\lim_{n\rightarrow\infty}\root\scriptstyle ^n\of {\frac 1 {(\ln n)^n}}$$ =0.

Then, by the root test my series converges absolutely.
 
  • #4
LCKurtz said:
What do you get when you simplify$$
\lim_{n\rightarrow\infty}\root\scriptstyle ^n\of {\frac 1 {(\ln n)^n}}$$Then what happens as ##n\rightarrow \infty##? Don't answer that by talking about some series test. This is just the limit of a sequence. What do you get?

knowLittle said:
$$
\lim_{n\rightarrow\infty}\root\scriptstyle ^n\of {\frac 1 {(\ln n)^n}}$$ =0.

Then, by the root test my series converges absolutely.

Do you have an aversion to showing your work? How did you get zero?
 
  • #5
##\lim _{n\rightarrow \infty }\left( \dfrac {1} {\ln n}\right) ^{\dfrac {n} {n}}=\lim _{n\rightarrow \infty }\dfrac {1} {\ln n}=0##
 
  • #6
Yes, thank you. So you see the root test was perfect for this problem. Quick and easy and instantly gives convergence. Nothing else needed.
 
  • #7
Thank you.
 

Related to Absolute Convergent, Conditionally Convergent?

What does it mean for a series to be absolutely convergent?

A series is said to be absolutely convergent if the sum of its terms converges to a finite value regardless of the order in which the terms are added. This means that even if the terms are rearranged, the series will still converge to the same value.

What is an example of an absolutely convergent series?

An example of an absolutely convergent series is the geometric series, where each term is multiplied by a common ratio. For example, 1 + 1/2 + 1/4 + 1/8 + ... is an absolutely convergent series with a sum of 2.

What is a conditionally convergent series?

A series is conditionally convergent if the sum of its terms converges, but the sum of the absolute values of its terms does not converge. This means that the order in which the terms are added can affect the overall sum of the series.

Can a series be both absolutely and conditionally convergent?

No, a series can either be absolutely convergent or conditionally convergent, but not both. If a series is absolutely convergent, it is also conditionally convergent, but the opposite is not true.

Why is it important to understand the difference between absolute and conditional convergence?

Understanding the difference between absolute and conditional convergence is important because it affects the behavior of series and their sums. Absolute convergence guarantees that the sum of a series will converge regardless of the order of its terms, while conditional convergence requires careful consideration of the order of terms to determine the sum. This distinction also has implications in real-world applications, such as in physics and engineering, where series are often used to model and predict real-world phenomena.

Similar threads

  • Calculus and Beyond Homework Help
Replies
2
Views
307
  • Calculus and Beyond Homework Help
Replies
1
Views
373
  • Calculus and Beyond Homework Help
Replies
2
Views
756
  • Calculus and Beyond Homework Help
Replies
4
Views
438
  • Calculus and Beyond Homework Help
Replies
34
Views
2K
  • Calculus and Beyond Homework Help
Replies
2
Views
806
  • Calculus and Beyond Homework Help
Replies
5
Views
1K
  • Calculus and Beyond Homework Help
Replies
4
Views
1K
  • Calculus and Beyond Homework Help
Replies
5
Views
1K
  • Calculus and Beyond Homework Help
Replies
7
Views
1K
Back
Top